LSAT and Law School Admissions Forum

Get expert LSAT preparation and law school admissions advice from PowerScore Test Preparation.

 Administrator
PowerScore Staff
  • PowerScore Staff
  • Posts: 8916
  • Joined: Feb 02, 2011
|
#22933
Complete Question Explanation

Assumption—CE. The correct answer choice is (C)

The doctor in this stimulus makes the following argument:
  • Premise 1: .....Blood pressure reduction medication often has harmful side effects.

    Premise 2: .....Diet and exercise works as well as medication.

    Conclusion: .....Therefore lifestyle changes are preferable to medication.
The author must presume that medication doesn't have other, greater benefits, and that lifestyle changes don't have more detrimental side effects.

Answer choice (A): The doctor does not argue that these are the only two methods that can be used, but rather compares only these two methods.

Answer choice (B): This is not an assumption required by the doctor's argument—the author provides that lifestyle changes are at least as effective as medication.

Answer choice (C): This is the correct answer choice, reflecting one of the prephrased answers in the discussion of the stimulus above. The doctor must presume that there aren't worse detriments associated with lifestyle changes.

Answer choice (D): This answer choice provides a broader principle than concluded by the doctor, and this is not an assumption required by the argument.

Answer choice (E): Since the author provides that the two methods are comparably effective, this answer choice does not provide an assumption on which the argument relies.
 Lina
  • Posts: 22
  • Joined: Aug 05, 2013
|
#10180
Hello, I took the Powerscore online course and am reviewing from my course book. I was hoping you could help me work out some problems from Chapter 5, Assumption

Pg 5-41 Q11: I selected answer choice A because I misunderstood to take the authors argument as the only option they see, so the language seemed like the definitive kind I should look for.


Thanks for your help!
 Steve Stein
PowerScore Staff
  • PowerScore Staff
  • Posts: 1153
  • Joined: Apr 11, 2011
|
#10185
Hi Lina,

In that one, doctor's conclusion is that it is healthier to rely on lifestyle changes to reduce blood pressure than on medication.

Such a comparison does not require assuming that those are the only two options, so answer choice A is not an assumption that the argument relies upon.

Answer choice C, however, does provide a necessary assumption: If the side effects that go with lifestyle changes are as bad as, or worse than, those of medication, the author's argument fails.

Since the author relies on the assumption presented in answer choice C, that is the right answer to that Assumption question.

I hope that's helpful--let me know whether this is clear--thanks!

~Steve
 Lina
  • Posts: 22
  • Joined: Aug 05, 2013
|
#10216
Thank you, this is very helpful!
 Lawyered
  • Posts: 23
  • Joined: Jun 13, 2017
|
#36134
How would we be able to ascertain D as being a broader than necessary explanation?

I was between C & D but D seemed to find a way to eliminate an alternate, better than C.
Administrator wrote:Complete Question Explanation

Assumption—CE. The correct answer choice is (C)

The doctor in this stimulus makes the following argument:
  • Premise 1: .....Blood pressure reduction medication often has harmful side effects.

    Premise 2: .....Diet and exercise works as well as medication.

    Conclusion: .....Therefore lifestyle changes are preferable to medication.
The author must presume that medication doesn't have other, greater benefits, and that lifestyle changes don't have more detrimental side effects.

Answer choice (A): The doctor does not argue that these are the only two methods that can be used, but rather compares only these two methods.

Answer choice (B): This is not an assumption required by the doctor's argument—the author provides that lifestyle changes are at least as effective as medication.

Answer choice (C): This is the correct answer choice, reflecting one of the prephrased answers in the discussion of the stimulus above. The doctor must presume that there aren't worse detriments associated with lifestyle changes.

Answer choice (D): This answer choice provides a broader principle than concluded by the doctor, and this is not an assumption required by the argument.

Answer choice (E): Since the author provides that the two methods are comparably effective, this answer choice does not provide an assumption on which the argument relies.
 Francis O'Rourke
PowerScore Staff
  • PowerScore Staff
  • Posts: 471
  • Joined: Mar 10, 2017
|
#36601
Hi Lawyered,

Choice (D) does strengthen the argument better than answer choice (C) does by eliminating any possible alternatives, but that is not necessary for this question, and generally looking for which answer choice will better guarantee the conclusion will be the wrong method to approach Assumption questions. We are looking for what is required, not what is strongest.

In general, you should be very suspicious of Assumption answer choices that have a universal scope. For example, answer choice (D) makes a claim about what is always true with all medications, all medication alternatives, and all medical conditions. As soon as you see such a broad implied scope to an answer choice, you should ask yourself whether the speaker truly needs to assume this about all medications.

In this case, the doctor is making a claim about blood pressure medication vs exercise and healthy diet. So, does the doctor need to believe something similar about arthritis or athlete's foot? If you choose answer choice (D) then you are in effect saying that for the doctor's argument to follow, she must believe similar things about the relative healthiness of other conditions.
 Tomars
  • Posts: 15
  • Joined: Aug 03, 2017
|
#38503
Francis O'Rourke wrote:Hi Lawyered,

Choice (D) does strengthen the argument better than answer choice (C) does by eliminating any possible alternatives, but that is not necessary for this question, and generally looking for which answer choice will better guarantee the conclusion will be the wrong method to approach Assumption questions. We are looking for what is required, not what is strongest.

In general, you should be very suspicious of Assumption answer choices that have a universal scope. For example, answer choice (D) makes a claim about what is always true with all medications, all medication alternatives, and all medical conditions. As soon as you see such a broad implied scope to an answer choice, you should ask yourself whether the speaker truly needs to assume this about all medications.

In this case, the doctor is making a claim about blood pressure medication vs exercise and healthy diet. So, does the doctor need to believe something similar about arthritis or athlete's foot? If you choose answer choice (D) then you are in effect saying that for the doctor's argument to follow, she must believe similar things about the relative healthiness of other conditions.

I'm curious about D and if would be a good sufficient assumption/justify the conclusion question. During review, I've been trying to see if I actually understand how a necessary vs sufficient answer choice would be presented.
 nicholaspavic
PowerScore Staff
  • PowerScore Staff
  • Posts: 271
  • Joined: Jun 12, 2017
|
#38802
Hi Tomars,
The language in Answer Option (D) is very strong and with a different justify question stem, then it may be a right answer. The language of Answer Option (D) is in fact so strong, that I think you could conceivably see this as a right answer choice in a justify scenario utilizing similar facts as you suggest. Part of the indication of a strong test-taker is her/his ability to recognize the applicability of other wrong answers to other types of questions. So well done! You are obviously mastering this material. :-D 8-)
 lunsandy
  • Posts: 61
  • Joined: Oct 14, 2017
|
#41014
Hi Powerscore,

I am still confused about why B is incorrect. In the stim we learn that life style changes are just as effective as taking medication. And then there's the jump to the conclusion that it is "healthier" (new word) to rely on these lifestyles changes to reduce BP. I chose B because it talks about the new word "healthier" and that it is supported by the stim "at least as effective."

Am I wrong because B doesn't address the assumption? Would this be correct if it was a MBT/ MSS? I am still confused why B is incorrect.

Thanks for the help!
 Francis O'Rourke
PowerScore Staff
  • PowerScore Staff
  • Posts: 471
  • Joined: Mar 10, 2017
|
#41125
Answer choice (B) begins with "If it is healthier to rely on lifestyle change than medication to reduce BP..." That claim is in fact the Doctor's conclusion. This should be a clear sign that this answer choice is not a necessary assumption for the argument. After all, why would it be necessary to add a consequence to the conclusion?

Looking at it another way, you can say that this answer choice is taking the Mistaken Reversal of the Doctor's assumption.

The doctor claimed that it is healthier to rely on lifestyle than medication because lifestyle change reduces blood pressure at least as effectively as medication
..... lifestyle at least as effective as medication :arrow: healthier to rely on lifestyle than on medication

Answer choice (B) reverses the relationship between the terms. It states that if it is healthier to rely on lifestyle, then lifestyle must be just as effective.
..... healthier to rely on lifestyle than on medication :arrow: lifestyle at least as effective as medication

This is neither necessary for the argument nor helpful. The doctor does not need to assume that the relationship between the evidence and conclusion is bi-conditional.

The test makers are experts at shifting around terms to create irrelevant information, so don't pick an answer merely because it uses the same terms as the stimulus. If you do this frequently, you will often fall victim to the Shell Game on answer choice such as this one.

Get the most out of your LSAT Prep Plus subscription.

Analyze and track your performance with our Testing and Analytics Package.